Đến nội dung

Dam Uoc Mo

Dam Uoc Mo

Đăng ký: 02-11-2013
Offline Đăng nhập: 30-12-2023 - 15:42
****-

Trong chủ đề: $1+2^{n}+4^{n}$ là số nguyên tố

14-11-2016 - 11:22

Bài 1: $2^{p}+2^{q}\vdots (pq). (*)$

Trường hợp 1: $p=q$

 

Thay vào (*) ta được $p^2 | 2.2^p$ suy ra $p=2$.

 

Trường hợp 2: $ p \ne q$.

Trường hợp 2.1: Một trong hai số $p,q$ bằng 2. Không mất tính tổng quát giả sử $q=2, p \ne 2$ Khi đó $2p |2^2+2^p \Rightarrow 2^2+2^p \equiv 0 (mod p)$ Mặt khác theo định lý Fermat, ta có:

$2^p \equiv 2 (mod p)$ $\Rightarrow 0 \equiv 2^2+2=2.3 (mod p)$ nên $p=3$. Kiểm tra trực tiếp thấy thỏa mãn vậy $(2,3), (3,2)$ thỏa mãn bài toán.

Trường hợp 2.2: $p \ne q, p,q \ne 2$. Theo định lí Fermat ta có:

$0 \equiv 2^p+2^q \equiv 2+2^q \equiv 2(1+2^{q-1}) (mod p) \Rightarrow p | 2(2^{q-1}+1) \Rightarrow p | 1+ 2^{q-1} (1) \Rightarrow 2^{2(q-1)} \equiv 1 (mod p)$

 

$\Rightarrow ord_p(2)| 2(q-1) \Rightarrow ord_p(2) | (2(q-1),p-1)=2(q-1,p-1)$. Do $p,q$ là hai số nguyên tố lẻ đặt:

 

$p-1=2^lm,q-1=2^kn$ trong đó $m, n$ là các số lẻ $k, l >0$. Khi đó $ord_p(2) | 2^{k+1}$ Như vậy $ord_p(2)=2^h ( 0 \le h \le k+1)$

 

Nếu $ 0 \le h \le k$ thì:  $((2^{2^h})^{2^{k-h}})^m \equiv 1 (mod p)$ hay $2^{q-1}-1 \equiv 0 (mod p)$ Mâu thuẫn với (1).

 

Vậy $ord_p(2)=2^{k+1} \Rightarrow 2^{k+1} | \varphi (p)=p-1=2^lm \Rightarrow 2^{k+1}|2^l \Rightarrow k<l$. Tương tự ta chứng minh được $l < k$ (vô lí). 

 

Kết luận $(p,q)=(2,2), (2,3), (3,2)$ thỏa mãn bài toán

 

Bài tương tự: Tìm $p,q$ nguyên tố sao cho $5^p+5^q \vdots pq$

Bài 2: bạn có thể xem lại đề không mình chỉ chứng minh được $n$ có dạng $3^k$ :(

Cho hỏi lại đoạn này tí, nhỡ nó chứa cả ước lẻ thì sao ?

Còn bài 2 liệu đặt n như thế có được không ?Nhỡ nó có ước nguyên tố khác thì sao? Mình cũng đọc có sách ghi vậy nên thắc mắc.


Trong chủ đề: $1+2^{n}+4^{n}$ là số nguyên tố

13-11-2016 - 23:30

Bài 1: $2^{p}+2^{q}\vdots (pq). (*)$

Trường hợp 1: $p=q$

 

Thay vào (*) ta được $p^2 | 2.2^p$ suy ra $p=2$.

 

Trường hợp 2: $ p \ne q$.

Trường hợp 2.1: Một trong hai số $p,q$ bằng 2. Không mất tính tổng quát giả sử $q=2, p \ne 2$ Khi đó $2p |2^2+2^p \Rightarrow 2^2+2^p \equiv 0 (mod p)$ Mặt khác theo định lý Fermat, ta có:

$2^p \equiv 2 (mod p)$ $\Rightarrow 0 \equiv 2^2+2=2.3 (mod p)$ nên $p=3$. Kiểm tra trực tiếp thấy thỏa mãn vậy $(2,3), (3,2)$ thỏa mãn bài toán.

Trường hợp 2.2: $p \ne q, p,q \ne 2$. Theo định lí Fermat ta có:

$0 \equiv 2^p+2^q \equiv 2+2^q \equiv 2(1+2^{q-1}) (mod p) \Rightarrow p | 2(2^{q-1}+1) \Rightarrow p | 1+ 2^{q-1} (1) \Rightarrow 2^{2(q-1)} \equiv 1 (mod p)$

 

$\Rightarrow ord_p(2)| 2(q-1) \Rightarrow ord_p(2) | (2(q-1),p-1)=2(q-1,p-1)$. Do $p,q$ là hai số nguyên tố lẻ đặt:

 

$p-1=2^lm,q-1=2^kn$ trong đó $m, n$ là các số lẻ $k, l >0$. Khi đó $ord_p(2) | 2^{k+1}$ Như vậy $ord_p(2)=2^h ( 0 \le h \le k+1)$

 

Nếu $ 0 \le h \le k$ thì:  $((2^{2^h})^{2^{k-h}})^m \equiv 1 (mod p)$ hay $2^{q-1}-1 \equiv 0 (mod p)$ Mâu thuẫn với (1).

 

Vậy $ord_p(2)=2^{k+1} \Rightarrow 2^{k+1} | \varphi (p)=p-1=2^lm \Rightarrow 2^{k+1}|2^l \Rightarrow k<l$. Tương tự ta chứng minh được $l < k$ (vô lí). 

 

Kết luận $(p,q)=(2,2), (2,3), (3,2)$ thỏa mãn bài toán

 

Bài tương tự: Tìm $p,q$ nguyên tố sao cho $5^p+5^q \vdots pq$

Bài 2: bạn có thể xem lại đề không mình chỉ chứng minh được $n$ có dạng $3^k$ :(

Hình như nó là chứng minh dạng đó, bài 2 ấy, nhưng đề mình bị sai nên ghi lẫn lộn, mình không hiểu nó nói gì nên đăng bài tổng quát lên đây nhờ các cao thủ. Chứng minh được $n=3^{k}$ là tìm được $n$ còn gì nữa. :)) Đấy bạn chứng minh giúp mình luôn được không? :D


Trong chủ đề: Cho số nguyên dương $a, n$ sao cho tất cả các ước nguyên tố của...

26-10-2016 - 23:09

Cái này chứng minh sao nhỉ $\frac{n-n/p^{s}}{p-1}\leq [ \frac{n}{p-1} ]$ ?

Chỗ đó có gì đâu.

$V_{p}(n!)< \frac{n}{p-1}\Rightarrow V_{p}(n!)\leq [\frac{n}{p-1}]$. 


Trong chủ đề: Cho số nguyên dương $a, n$ sao cho tất cả các ước nguyên tố của...

25-10-2016 - 21:42

Cho số nguyên dương $a, n$ sao cho tất cả các ước nguyên tố của $a$ đều lớn hơn $n.$ Chứng minh rằng $(a-1)(a^{2}-1)...(a^{n-1}-1)$ chia hết cho $n!.$

Gọi $p$ là 1 ước nguyên tố của $n$.

Có $a^{p-1}\equiv 1(modp)$ nên $V_{p}(VT)\geq V_{p}(\prod_{i=1}^{[\frac{n}{p-1}]}(a^{(p-1)i}-1))\geq [\frac{n}{p-1}] \\ V_{p}(n!)=[n/p]+[n/p^{2}]+...[n/p^{s}]\leq n/p+n/p^{2}+...+n/p^{s}=\frac{n-n/p^{s}}{p-1}\leq [\frac{n}{p-1}] \\ --> Q.E.D.\blacksquare$

P.S: Lâu lắm mới làm bài. Mỏi quá. :))


Trong chủ đề: $(2^{n}+1)\vdots n^{2}$

07-09-2016 - 21:56

Mình nghĩ 2 cách giải trên đã có nhầm lẫn, mình sẽ tiếp tục ý tưởng như này: $2^{n}+1\equiv 0(mod n^{2})\\ --> (-8)^{l}\equiv 1(mod 9l^{2})\\ -->(-8)^{l}\equiv 1(modl)$. Đến đây có thể gọi m là ước nguyên tố bé nhất của l rồi dùng bậc của số nguyên để giải quyết nốt. :D